Hi LT! Thanks for your question, and happy to help
Answer choice (C) is not necessarily a weaker answer. What answer choice (C) does to strengthen the conclusion is it defends the conclusion against a possible objection to the argument. To see why, let's break down the argument.
The conclusion of the argument is that democratic societies should adopt policies that ensure constant economic expansion. The reason the author offers to support this conclusion is that economic expansion allows people to improve their economic standing. But what if the rich just get richer and those who are not rich do not really improve their economic standing? If that was the case, then the wide income gap that the author discusses as dangerous may get worse, meaning that democratic societies shouldn't adopt policies that ensure constant economic expansion because it may endanger society further.
Answer choice (C) tells us that economic expansion will actually help those at low income levels more so than those at other income levels. So the concern that we pointed out when examining the argument is not actually a concern! If economic expansion helps those at low income levels most with improving their economic standing, then economic expansion is a good thing for democratic societies because it protects against this class segmentation that has negative effects by preventing wide income gaps.
Therefore, answer choice (C) is the correct answer choice because it defends the argument against a potential weakness. Defending an argument against its weakness is one way to strengthen an argument. This sort of correct answer choice is not necessarily a weaker answer; it just strengthens the conclusion in a different way from an answer choice that actively strengthens the argument by adding a piece of information to support the reasoning. This type of answer choice is similar to Defender Assumption answer choices for Assumption questions; Defender Assumption answer choices aren't weaker than Supporter Assumption answer choices, but they just support the argument in a different way.
I hope this helps, and let me know if you have any other questions!